Exponents of ec primes ending with digit 7











up vote
0
down vote

favorite












A conjecture about numbers of the form $10^{m}(2^{k}−1)+2^{k-1}−1$, where $m$ is the number of decimal digits of $ 2^{k-1}$.
In this question ec-numbers are introduced, obttained by the concatenation of two consecutive Mersenne numbers (12763=ec(7) for example). The values of $k$ for which $ec(k)$ is prime are: $[2, 3, 4, 7, 8, 12, 19, 22, 36, 46, 51, 67, 79, 215, 359, 394, 451, 1323, 2131, 3336, 3371, 6231, 19179, 39699, 51456, 56238, 69660, 75894, 79798, 92020, 174968, 176006, 181015, 285019, 331259, 360787, 366770]$
$7, 67, 360787$ are all the $k's$ leading to a prime ending with digit $7$. $7,67,360787$ are all congruent to $1$ $pmod 6$. My question is: does a k exist such that k ends with digit $7$, $ec(k)$ is prime and $k$ is NOT congruent to 1 $pmod 6$? Or that can be ruled out? In other words, if $k$ is congruent to 7 mod 10 and $ec(k)$ is prime, then necessarly $k$ must be congruent to 1 mod 6?










share|cite|improve this question
























  • I noticed that the largest factor of $6,66,360786$ is a palindrome. The largest factor of 6 is 3, the largest factor of 66 is 11, the largest factor of 360786 is 383.
    – paolo galli
    Nov 19 at 7:51










  • @Giovanni Resta can you find a counter-example?
    – paolo galli
    Nov 19 at 7:52










  • @Peter i think it is hard to find a counterexample, maybe it can be ruled out with a proof.
    – paolo galli
    Nov 19 at 8:15















up vote
0
down vote

favorite












A conjecture about numbers of the form $10^{m}(2^{k}−1)+2^{k-1}−1$, where $m$ is the number of decimal digits of $ 2^{k-1}$.
In this question ec-numbers are introduced, obttained by the concatenation of two consecutive Mersenne numbers (12763=ec(7) for example). The values of $k$ for which $ec(k)$ is prime are: $[2, 3, 4, 7, 8, 12, 19, 22, 36, 46, 51, 67, 79, 215, 359, 394, 451, 1323, 2131, 3336, 3371, 6231, 19179, 39699, 51456, 56238, 69660, 75894, 79798, 92020, 174968, 176006, 181015, 285019, 331259, 360787, 366770]$
$7, 67, 360787$ are all the $k's$ leading to a prime ending with digit $7$. $7,67,360787$ are all congruent to $1$ $pmod 6$. My question is: does a k exist such that k ends with digit $7$, $ec(k)$ is prime and $k$ is NOT congruent to 1 $pmod 6$? Or that can be ruled out? In other words, if $k$ is congruent to 7 mod 10 and $ec(k)$ is prime, then necessarly $k$ must be congruent to 1 mod 6?










share|cite|improve this question
























  • I noticed that the largest factor of $6,66,360786$ is a palindrome. The largest factor of 6 is 3, the largest factor of 66 is 11, the largest factor of 360786 is 383.
    – paolo galli
    Nov 19 at 7:51










  • @Giovanni Resta can you find a counter-example?
    – paolo galli
    Nov 19 at 7:52










  • @Peter i think it is hard to find a counterexample, maybe it can be ruled out with a proof.
    – paolo galli
    Nov 19 at 8:15













up vote
0
down vote

favorite









up vote
0
down vote

favorite











A conjecture about numbers of the form $10^{m}(2^{k}−1)+2^{k-1}−1$, where $m$ is the number of decimal digits of $ 2^{k-1}$.
In this question ec-numbers are introduced, obttained by the concatenation of two consecutive Mersenne numbers (12763=ec(7) for example). The values of $k$ for which $ec(k)$ is prime are: $[2, 3, 4, 7, 8, 12, 19, 22, 36, 46, 51, 67, 79, 215, 359, 394, 451, 1323, 2131, 3336, 3371, 6231, 19179, 39699, 51456, 56238, 69660, 75894, 79798, 92020, 174968, 176006, 181015, 285019, 331259, 360787, 366770]$
$7, 67, 360787$ are all the $k's$ leading to a prime ending with digit $7$. $7,67,360787$ are all congruent to $1$ $pmod 6$. My question is: does a k exist such that k ends with digit $7$, $ec(k)$ is prime and $k$ is NOT congruent to 1 $pmod 6$? Or that can be ruled out? In other words, if $k$ is congruent to 7 mod 10 and $ec(k)$ is prime, then necessarly $k$ must be congruent to 1 mod 6?










share|cite|improve this question















A conjecture about numbers of the form $10^{m}(2^{k}−1)+2^{k-1}−1$, where $m$ is the number of decimal digits of $ 2^{k-1}$.
In this question ec-numbers are introduced, obttained by the concatenation of two consecutive Mersenne numbers (12763=ec(7) for example). The values of $k$ for which $ec(k)$ is prime are: $[2, 3, 4, 7, 8, 12, 19, 22, 36, 46, 51, 67, 79, 215, 359, 394, 451, 1323, 2131, 3336, 3371, 6231, 19179, 39699, 51456, 56238, 69660, 75894, 79798, 92020, 174968, 176006, 181015, 285019, 331259, 360787, 366770]$
$7, 67, 360787$ are all the $k's$ leading to a prime ending with digit $7$. $7,67,360787$ are all congruent to $1$ $pmod 6$. My question is: does a k exist such that k ends with digit $7$, $ec(k)$ is prime and $k$ is NOT congruent to 1 $pmod 6$? Or that can be ruled out? In other words, if $k$ is congruent to 7 mod 10 and $ec(k)$ is prime, then necessarly $k$ must be congruent to 1 mod 6?







number-theory






share|cite|improve this question















share|cite|improve this question













share|cite|improve this question




share|cite|improve this question








edited Nov 19 at 7:31

























asked Nov 19 at 6:58









paolo galli

223




223












  • I noticed that the largest factor of $6,66,360786$ is a palindrome. The largest factor of 6 is 3, the largest factor of 66 is 11, the largest factor of 360786 is 383.
    – paolo galli
    Nov 19 at 7:51










  • @Giovanni Resta can you find a counter-example?
    – paolo galli
    Nov 19 at 7:52










  • @Peter i think it is hard to find a counterexample, maybe it can be ruled out with a proof.
    – paolo galli
    Nov 19 at 8:15


















  • I noticed that the largest factor of $6,66,360786$ is a palindrome. The largest factor of 6 is 3, the largest factor of 66 is 11, the largest factor of 360786 is 383.
    – paolo galli
    Nov 19 at 7:51










  • @Giovanni Resta can you find a counter-example?
    – paolo galli
    Nov 19 at 7:52










  • @Peter i think it is hard to find a counterexample, maybe it can be ruled out with a proof.
    – paolo galli
    Nov 19 at 8:15
















I noticed that the largest factor of $6,66,360786$ is a palindrome. The largest factor of 6 is 3, the largest factor of 66 is 11, the largest factor of 360786 is 383.
– paolo galli
Nov 19 at 7:51




I noticed that the largest factor of $6,66,360786$ is a palindrome. The largest factor of 6 is 3, the largest factor of 66 is 11, the largest factor of 360786 is 383.
– paolo galli
Nov 19 at 7:51












@Giovanni Resta can you find a counter-example?
– paolo galli
Nov 19 at 7:52




@Giovanni Resta can you find a counter-example?
– paolo galli
Nov 19 at 7:52












@Peter i think it is hard to find a counterexample, maybe it can be ruled out with a proof.
– paolo galli
Nov 19 at 8:15




@Peter i think it is hard to find a counterexample, maybe it can be ruled out with a proof.
– paolo galli
Nov 19 at 8:15















active

oldest

votes











Your Answer





StackExchange.ifUsing("editor", function () {
return StackExchange.using("mathjaxEditing", function () {
StackExchange.MarkdownEditor.creationCallbacks.add(function (editor, postfix) {
StackExchange.mathjaxEditing.prepareWmdForMathJax(editor, postfix, [["$", "$"], ["\\(","\\)"]]);
});
});
}, "mathjax-editing");

StackExchange.ready(function() {
var channelOptions = {
tags: "".split(" "),
id: "69"
};
initTagRenderer("".split(" "), "".split(" "), channelOptions);

StackExchange.using("externalEditor", function() {
// Have to fire editor after snippets, if snippets enabled
if (StackExchange.settings.snippets.snippetsEnabled) {
StackExchange.using("snippets", function() {
createEditor();
});
}
else {
createEditor();
}
});

function createEditor() {
StackExchange.prepareEditor({
heartbeatType: 'answer',
convertImagesToLinks: true,
noModals: true,
showLowRepImageUploadWarning: true,
reputationToPostImages: 10,
bindNavPrevention: true,
postfix: "",
imageUploader: {
brandingHtml: "Powered by u003ca class="icon-imgur-white" href="https://imgur.com/"u003eu003c/au003e",
contentPolicyHtml: "User contributions licensed under u003ca href="https://creativecommons.org/licenses/by-sa/3.0/"u003ecc by-sa 3.0 with attribution requiredu003c/au003e u003ca href="https://stackoverflow.com/legal/content-policy"u003e(content policy)u003c/au003e",
allowUrls: true
},
noCode: true, onDemand: true,
discardSelector: ".discard-answer"
,immediatelyShowMarkdownHelp:true
});


}
});














draft saved

draft discarded


















StackExchange.ready(
function () {
StackExchange.openid.initPostLogin('.new-post-login', 'https%3a%2f%2fmath.stackexchange.com%2fquestions%2f3004600%2fexponents-of-ec-primes-ending-with-digit-7%23new-answer', 'question_page');
}
);

Post as a guest















Required, but never shown






























active

oldest

votes













active

oldest

votes









active

oldest

votes






active

oldest

votes
















draft saved

draft discarded




















































Thanks for contributing an answer to Mathematics Stack Exchange!


  • Please be sure to answer the question. Provide details and share your research!

But avoid



  • Asking for help, clarification, or responding to other answers.

  • Making statements based on opinion; back them up with references or personal experience.


Use MathJax to format equations. MathJax reference.


To learn more, see our tips on writing great answers.





Some of your past answers have not been well-received, and you're in danger of being blocked from answering.


Please pay close attention to the following guidance:


  • Please be sure to answer the question. Provide details and share your research!

But avoid



  • Asking for help, clarification, or responding to other answers.

  • Making statements based on opinion; back them up with references or personal experience.


To learn more, see our tips on writing great answers.




draft saved


draft discarded














StackExchange.ready(
function () {
StackExchange.openid.initPostLogin('.new-post-login', 'https%3a%2f%2fmath.stackexchange.com%2fquestions%2f3004600%2fexponents-of-ec-primes-ending-with-digit-7%23new-answer', 'question_page');
}
);

Post as a guest















Required, but never shown





















































Required, but never shown














Required, but never shown












Required, but never shown







Required, but never shown

































Required, but never shown














Required, but never shown












Required, but never shown







Required, but never shown







Popular posts from this blog

Biblatex bibliography style without URLs when DOI exists (in Overleaf with Zotero bibliography)

ComboBox Display Member on multiple fields

Is it possible to collect Nectar points via Trainline?